zainrizvi
Thanks Received: 16
Atticus Finch
Atticus Finch
 
Posts: 171
Joined: July 19th, 2011
 
 
trophy
First Responder
 

Q14 - The plant manager's argument is most

by zainrizvi Wed Jul 27, 2011 7:21 pm

Can anyone explain why (A) is the correct choice? I thought that since the conclusion deals with both costs and profits(net effect), whereas the evidence only deals with costs(some of the factors), the answer makes sense. But I just wanted to confirm if this line of reasoning is correct.
 
giladedelman
Thanks Received: 833
LSAT Geek
 
Posts: 619
Joined: April 04th, 2010
 
This post thanked 4 times.
 
 

Re: Q14 - The plant manager's argument is most

by giladedelman Sat Jul 30, 2011 2:34 pm

You've got it exactly right. The plant manager introduces evidence showing that certain costs would increase. But this doesn't tell us anything about 1) any savings that the switch might generate elsewhere or 2) any revenue increases the switch might lead to. That's why (A) is correct; you can't make a conclusion about net effects without taking both costs and revenues/savings into account.

(B) is incorrect because the evidence actually is verifiable; it's about costs, not just some vague appeals to authority.

(C) is incorrect because the conclusion does not repeat the evidence.

(D) is out because there is no such probable-to-definite movement going on here. Everything is expressed with the same degree of certainty.

(E) is incorrect because while the evidence is incomplete, it is certainly relevant.

Thanks for posting!
 
Dtodaizzle
Thanks Received: 0
Vinny Gambini
Vinny Gambini
 
Posts: 24
Joined: February 08th, 2015
 
 
 

Re: Q14 - The plant manager's argument is most

by Dtodaizzle Mon Sep 21, 2015 10:30 am

I still don’t see why B is also a potential right answer. The cost savings that the plant manager are based on claims he made; he doesn’t cite any sources on where he got the information that the new process cost more to install/run.

It is definitely possible to check whether or not the claims that the manager made are true or not, but why can’t the effort of verifying the manager’s claims be construed as a weakness for this argument?
User avatar
 
ohthatpatrick
Thanks Received: 3808
Atticus Finch
Atticus Finch
 
Posts: 4661
Joined: April 01st, 2011
 
 
 

Re: Q14 - The plant manager's argument is most

by ohthatpatrick Mon Sep 28, 2015 6:44 pm

The short answer to this is that (B) would probably NEVER be a flaw on LSAT. It doesn't represent a 'reasoning' flaw.

A reasoning flaw is a problem with the move from Premise to Conclusion: i.e. one can accept the Evidence as true but still dispute the truth of the Conclusion.

(B) feels like you're attacking the truth of the Evidence. That's just not how LSAT works.

I don't want to say (B) is impossible as a flaw, but it's incredibly unlikely on LSAT and I mostly want to warn you that it SHOULD feel like an unfair line of criticism, given the way the test works.

If the plant manager said:
Everybody can be certain that Joe stole the donut. After all, I saw him steal the donut.

That is probably vulnerable to (B)'s criticism. There's no way to independently verify whether the plant manager really did see Joe steal a donut (unless there's some security camera footage laying around somewhere). :)

You were saying that (B) is accurate for this argument because the plant manager does not cite and sources for his information.

But the keyword in (B) is whether the plant manager's claims are "verifiable". COULD these claims be verified, not HAVE they been.

Since his claims are quantitative in nature, they COULD be independently verified. A neutral party could verify that the new equipment is expensive to buy and install, etc.

So the support for the conclusion DOES include verifiable evidence.

Hope this helps.